If $M/Ncong M$, can we conclude that $N=0$? [duplicate]












1















This question already has an answer here:




  • Is there a (f.g., free) module isomorphic to a quotient of itself?

    4 answers




Let $M$ be an $R$-module, where we may assume that $R$ is an integral domain.



Let $N$ be a submodule of $M$.



Suppose that $M/Ncong M$. Can we conclude that $N=0$?



(If no, what are some sufficient conditions that make it true?)



Update: I learn that for "infinite dimensional" cases it can fail. How about when $M$ is finitely generated, does it work?



Thanks a lot.










share|cite|improve this question















marked as duplicate by rschwieb abstract-algebra
Users with the  abstract-algebra badge can single-handedly close abstract-algebra questions as duplicates and reopen them as needed.

StackExchange.ready(function() {
if (StackExchange.options.isMobile) return;

$('.dupe-hammer-message-hover:not(.hover-bound)').each(function() {
var $hover = $(this).addClass('hover-bound'),
$msg = $hover.siblings('.dupe-hammer-message');

$hover.hover(
function() {
$hover.showInfoMessage('', {
messageElement: $msg.clone().show(),
transient: false,
position: { my: 'bottom left', at: 'top center', offsetTop: -7 },
dismissable: false,
relativeToBody: true
});
},
function() {
StackExchange.helpers.removeMessages();
}
);
});
});
Nov 21 '18 at 15:07


This question has been asked before and already has an answer. If those answers do not fully address your question, please ask a new question.















  • This is false, but see statement 3 of Nakayama's Lemma on wikipedia.
    – Aaron
    Nov 21 '18 at 5:10










  • how about when $M$ is finitely generated, does it work? Nope. You can get $Roplus Rcong R$ as $R$ modules for some rings. This is clearly finitely generated with a quotient by a nonzero ideal isororphic to $R$. See egreg's answer in the linked duplicate.
    – rschwieb
    Nov 21 '18 at 15:05


















1















This question already has an answer here:




  • Is there a (f.g., free) module isomorphic to a quotient of itself?

    4 answers




Let $M$ be an $R$-module, where we may assume that $R$ is an integral domain.



Let $N$ be a submodule of $M$.



Suppose that $M/Ncong M$. Can we conclude that $N=0$?



(If no, what are some sufficient conditions that make it true?)



Update: I learn that for "infinite dimensional" cases it can fail. How about when $M$ is finitely generated, does it work?



Thanks a lot.










share|cite|improve this question















marked as duplicate by rschwieb abstract-algebra
Users with the  abstract-algebra badge can single-handedly close abstract-algebra questions as duplicates and reopen them as needed.

StackExchange.ready(function() {
if (StackExchange.options.isMobile) return;

$('.dupe-hammer-message-hover:not(.hover-bound)').each(function() {
var $hover = $(this).addClass('hover-bound'),
$msg = $hover.siblings('.dupe-hammer-message');

$hover.hover(
function() {
$hover.showInfoMessage('', {
messageElement: $msg.clone().show(),
transient: false,
position: { my: 'bottom left', at: 'top center', offsetTop: -7 },
dismissable: false,
relativeToBody: true
});
},
function() {
StackExchange.helpers.removeMessages();
}
);
});
});
Nov 21 '18 at 15:07


This question has been asked before and already has an answer. If those answers do not fully address your question, please ask a new question.















  • This is false, but see statement 3 of Nakayama's Lemma on wikipedia.
    – Aaron
    Nov 21 '18 at 5:10










  • how about when $M$ is finitely generated, does it work? Nope. You can get $Roplus Rcong R$ as $R$ modules for some rings. This is clearly finitely generated with a quotient by a nonzero ideal isororphic to $R$. See egreg's answer in the linked duplicate.
    – rschwieb
    Nov 21 '18 at 15:05
















1












1








1


1






This question already has an answer here:




  • Is there a (f.g., free) module isomorphic to a quotient of itself?

    4 answers




Let $M$ be an $R$-module, where we may assume that $R$ is an integral domain.



Let $N$ be a submodule of $M$.



Suppose that $M/Ncong M$. Can we conclude that $N=0$?



(If no, what are some sufficient conditions that make it true?)



Update: I learn that for "infinite dimensional" cases it can fail. How about when $M$ is finitely generated, does it work?



Thanks a lot.










share|cite|improve this question
















This question already has an answer here:




  • Is there a (f.g., free) module isomorphic to a quotient of itself?

    4 answers




Let $M$ be an $R$-module, where we may assume that $R$ is an integral domain.



Let $N$ be a submodule of $M$.



Suppose that $M/Ncong M$. Can we conclude that $N=0$?



(If no, what are some sufficient conditions that make it true?)



Update: I learn that for "infinite dimensional" cases it can fail. How about when $M$ is finitely generated, does it work?



Thanks a lot.





This question already has an answer here:




  • Is there a (f.g., free) module isomorphic to a quotient of itself?

    4 answers








abstract-algebra modules






share|cite|improve this question















share|cite|improve this question













share|cite|improve this question




share|cite|improve this question








edited Nov 21 '18 at 4:00

























asked Nov 21 '18 at 3:51









yoyostein

7,89293768




7,89293768




marked as duplicate by rschwieb abstract-algebra
Users with the  abstract-algebra badge can single-handedly close abstract-algebra questions as duplicates and reopen them as needed.

StackExchange.ready(function() {
if (StackExchange.options.isMobile) return;

$('.dupe-hammer-message-hover:not(.hover-bound)').each(function() {
var $hover = $(this).addClass('hover-bound'),
$msg = $hover.siblings('.dupe-hammer-message');

$hover.hover(
function() {
$hover.showInfoMessage('', {
messageElement: $msg.clone().show(),
transient: false,
position: { my: 'bottom left', at: 'top center', offsetTop: -7 },
dismissable: false,
relativeToBody: true
});
},
function() {
StackExchange.helpers.removeMessages();
}
);
});
});
Nov 21 '18 at 15:07


This question has been asked before and already has an answer. If those answers do not fully address your question, please ask a new question.






marked as duplicate by rschwieb abstract-algebra
Users with the  abstract-algebra badge can single-handedly close abstract-algebra questions as duplicates and reopen them as needed.

StackExchange.ready(function() {
if (StackExchange.options.isMobile) return;

$('.dupe-hammer-message-hover:not(.hover-bound)').each(function() {
var $hover = $(this).addClass('hover-bound'),
$msg = $hover.siblings('.dupe-hammer-message');

$hover.hover(
function() {
$hover.showInfoMessage('', {
messageElement: $msg.clone().show(),
transient: false,
position: { my: 'bottom left', at: 'top center', offsetTop: -7 },
dismissable: false,
relativeToBody: true
});
},
function() {
StackExchange.helpers.removeMessages();
}
);
});
});
Nov 21 '18 at 15:07


This question has been asked before and already has an answer. If those answers do not fully address your question, please ask a new question.














  • This is false, but see statement 3 of Nakayama's Lemma on wikipedia.
    – Aaron
    Nov 21 '18 at 5:10










  • how about when $M$ is finitely generated, does it work? Nope. You can get $Roplus Rcong R$ as $R$ modules for some rings. This is clearly finitely generated with a quotient by a nonzero ideal isororphic to $R$. See egreg's answer in the linked duplicate.
    – rschwieb
    Nov 21 '18 at 15:05




















  • This is false, but see statement 3 of Nakayama's Lemma on wikipedia.
    – Aaron
    Nov 21 '18 at 5:10










  • how about when $M$ is finitely generated, does it work? Nope. You can get $Roplus Rcong R$ as $R$ modules for some rings. This is clearly finitely generated with a quotient by a nonzero ideal isororphic to $R$. See egreg's answer in the linked duplicate.
    – rschwieb
    Nov 21 '18 at 15:05


















This is false, but see statement 3 of Nakayama's Lemma on wikipedia.
– Aaron
Nov 21 '18 at 5:10




This is false, but see statement 3 of Nakayama's Lemma on wikipedia.
– Aaron
Nov 21 '18 at 5:10












how about when $M$ is finitely generated, does it work? Nope. You can get $Roplus Rcong R$ as $R$ modules for some rings. This is clearly finitely generated with a quotient by a nonzero ideal isororphic to $R$. See egreg's answer in the linked duplicate.
– rschwieb
Nov 21 '18 at 15:05






how about when $M$ is finitely generated, does it work? Nope. You can get $Roplus Rcong R$ as $R$ modules for some rings. This is clearly finitely generated with a quotient by a nonzero ideal isororphic to $R$. See egreg's answer in the linked duplicate.
– rschwieb
Nov 21 '18 at 15:05












2 Answers
2






active

oldest

votes


















2














It fails for vector spaces of infinite dimension.






share|cite|improve this answer





























    2














    It is not true. Let $M = bigoplus_{n=1}^infty mathbb{Z}$, and let $N = {(z,0,0,0,ldots),|, z in mathbb{Z}}$. Then $M/N cong M$.






    share|cite|improve this answer





















    • How about when M is finitely generated, does it work?
      – yoyostein
      Nov 21 '18 at 4:00






    • 1




      If $M$ is a vector space, then yes it does. If $M$ is not a vector space, then I'm not so sure...
      – Monstrous Moonshiner
      Nov 21 '18 at 4:00






    • 2




      It still holds if we let $R = bigoplus^infty mathbb Z$, while $M=R$, and $N$ stays the same. I believe $R$ and $M$ need to be Noetherian for that to hold.
      – Quang Hoang
      Nov 21 '18 at 4:07




















    2 Answers
    2






    active

    oldest

    votes








    2 Answers
    2






    active

    oldest

    votes









    active

    oldest

    votes






    active

    oldest

    votes









    2














    It fails for vector spaces of infinite dimension.






    share|cite|improve this answer


























      2














      It fails for vector spaces of infinite dimension.






      share|cite|improve this answer
























        2












        2








        2






        It fails for vector spaces of infinite dimension.






        share|cite|improve this answer












        It fails for vector spaces of infinite dimension.







        share|cite|improve this answer












        share|cite|improve this answer



        share|cite|improve this answer










        answered Nov 21 '18 at 3:55









        Dante Grevino

        94319




        94319























            2














            It is not true. Let $M = bigoplus_{n=1}^infty mathbb{Z}$, and let $N = {(z,0,0,0,ldots),|, z in mathbb{Z}}$. Then $M/N cong M$.






            share|cite|improve this answer





















            • How about when M is finitely generated, does it work?
              – yoyostein
              Nov 21 '18 at 4:00






            • 1




              If $M$ is a vector space, then yes it does. If $M$ is not a vector space, then I'm not so sure...
              – Monstrous Moonshiner
              Nov 21 '18 at 4:00






            • 2




              It still holds if we let $R = bigoplus^infty mathbb Z$, while $M=R$, and $N$ stays the same. I believe $R$ and $M$ need to be Noetherian for that to hold.
              – Quang Hoang
              Nov 21 '18 at 4:07


















            2














            It is not true. Let $M = bigoplus_{n=1}^infty mathbb{Z}$, and let $N = {(z,0,0,0,ldots),|, z in mathbb{Z}}$. Then $M/N cong M$.






            share|cite|improve this answer





















            • How about when M is finitely generated, does it work?
              – yoyostein
              Nov 21 '18 at 4:00






            • 1




              If $M$ is a vector space, then yes it does. If $M$ is not a vector space, then I'm not so sure...
              – Monstrous Moonshiner
              Nov 21 '18 at 4:00






            • 2




              It still holds if we let $R = bigoplus^infty mathbb Z$, while $M=R$, and $N$ stays the same. I believe $R$ and $M$ need to be Noetherian for that to hold.
              – Quang Hoang
              Nov 21 '18 at 4:07
















            2












            2








            2






            It is not true. Let $M = bigoplus_{n=1}^infty mathbb{Z}$, and let $N = {(z,0,0,0,ldots),|, z in mathbb{Z}}$. Then $M/N cong M$.






            share|cite|improve this answer












            It is not true. Let $M = bigoplus_{n=1}^infty mathbb{Z}$, and let $N = {(z,0,0,0,ldots),|, z in mathbb{Z}}$. Then $M/N cong M$.







            share|cite|improve this answer












            share|cite|improve this answer



            share|cite|improve this answer










            answered Nov 21 '18 at 3:56









            Monstrous Moonshiner

            2,25011337




            2,25011337












            • How about when M is finitely generated, does it work?
              – yoyostein
              Nov 21 '18 at 4:00






            • 1




              If $M$ is a vector space, then yes it does. If $M$ is not a vector space, then I'm not so sure...
              – Monstrous Moonshiner
              Nov 21 '18 at 4:00






            • 2




              It still holds if we let $R = bigoplus^infty mathbb Z$, while $M=R$, and $N$ stays the same. I believe $R$ and $M$ need to be Noetherian for that to hold.
              – Quang Hoang
              Nov 21 '18 at 4:07




















            • How about when M is finitely generated, does it work?
              – yoyostein
              Nov 21 '18 at 4:00






            • 1




              If $M$ is a vector space, then yes it does. If $M$ is not a vector space, then I'm not so sure...
              – Monstrous Moonshiner
              Nov 21 '18 at 4:00






            • 2




              It still holds if we let $R = bigoplus^infty mathbb Z$, while $M=R$, and $N$ stays the same. I believe $R$ and $M$ need to be Noetherian for that to hold.
              – Quang Hoang
              Nov 21 '18 at 4:07


















            How about when M is finitely generated, does it work?
            – yoyostein
            Nov 21 '18 at 4:00




            How about when M is finitely generated, does it work?
            – yoyostein
            Nov 21 '18 at 4:00




            1




            1




            If $M$ is a vector space, then yes it does. If $M$ is not a vector space, then I'm not so sure...
            – Monstrous Moonshiner
            Nov 21 '18 at 4:00




            If $M$ is a vector space, then yes it does. If $M$ is not a vector space, then I'm not so sure...
            – Monstrous Moonshiner
            Nov 21 '18 at 4:00




            2




            2




            It still holds if we let $R = bigoplus^infty mathbb Z$, while $M=R$, and $N$ stays the same. I believe $R$ and $M$ need to be Noetherian for that to hold.
            – Quang Hoang
            Nov 21 '18 at 4:07






            It still holds if we let $R = bigoplus^infty mathbb Z$, while $M=R$, and $N$ stays the same. I believe $R$ and $M$ need to be Noetherian for that to hold.
            – Quang Hoang
            Nov 21 '18 at 4:07





            Popular posts from this blog

            Can a sorcerer learn a 5th-level spell early by creating spell slots using the Font of Magic feature?

            Does disintegrating a polymorphed enemy still kill it after the 2018 errata?

            A Topological Invariant for $pi_3(U(n))$